Linear Programming Negativity Constraints












0












$begingroup$


What happens when a variable is negative?



An example would be:



Maximize z = 3x1 + 4x2, subject to constraints:




  1. 2x1 + 3x2 <= 10

  2. 2x1 - 4x2 <= 20

  3. x2 <= 10

  4. x1 >= 0


To set up an Linear Programming problem in Standard Form, I learned that it must be of maximization type. The constraints must be <= (which is good as 1) and 2) agree with that). However, x1 is >= 0, but x2 is not. What would one do in this case? I tried introducing slack variables, namely x2', but I don't know where to go from here, any hints/help would be appreciated. I am just confused on what to do when one of the variables does not satisfy the positivity constraint, here x2<=10.



Edit: I think I can rewrite x2<= 10 as x2-10 <= 0. Then introduce x2' = x2-10. Then, can I replace x2 with x2'+10?










share|cite|improve this question











$endgroup$

















    0












    $begingroup$


    What happens when a variable is negative?



    An example would be:



    Maximize z = 3x1 + 4x2, subject to constraints:




    1. 2x1 + 3x2 <= 10

    2. 2x1 - 4x2 <= 20

    3. x2 <= 10

    4. x1 >= 0


    To set up an Linear Programming problem in Standard Form, I learned that it must be of maximization type. The constraints must be <= (which is good as 1) and 2) agree with that). However, x1 is >= 0, but x2 is not. What would one do in this case? I tried introducing slack variables, namely x2', but I don't know where to go from here, any hints/help would be appreciated. I am just confused on what to do when one of the variables does not satisfy the positivity constraint, here x2<=10.



    Edit: I think I can rewrite x2<= 10 as x2-10 <= 0. Then introduce x2' = x2-10. Then, can I replace x2 with x2'+10?










    share|cite|improve this question











    $endgroup$















      0












      0








      0





      $begingroup$


      What happens when a variable is negative?



      An example would be:



      Maximize z = 3x1 + 4x2, subject to constraints:




      1. 2x1 + 3x2 <= 10

      2. 2x1 - 4x2 <= 20

      3. x2 <= 10

      4. x1 >= 0


      To set up an Linear Programming problem in Standard Form, I learned that it must be of maximization type. The constraints must be <= (which is good as 1) and 2) agree with that). However, x1 is >= 0, but x2 is not. What would one do in this case? I tried introducing slack variables, namely x2', but I don't know where to go from here, any hints/help would be appreciated. I am just confused on what to do when one of the variables does not satisfy the positivity constraint, here x2<=10.



      Edit: I think I can rewrite x2<= 10 as x2-10 <= 0. Then introduce x2' = x2-10. Then, can I replace x2 with x2'+10?










      share|cite|improve this question











      $endgroup$




      What happens when a variable is negative?



      An example would be:



      Maximize z = 3x1 + 4x2, subject to constraints:




      1. 2x1 + 3x2 <= 10

      2. 2x1 - 4x2 <= 20

      3. x2 <= 10

      4. x1 >= 0


      To set up an Linear Programming problem in Standard Form, I learned that it must be of maximization type. The constraints must be <= (which is good as 1) and 2) agree with that). However, x1 is >= 0, but x2 is not. What would one do in this case? I tried introducing slack variables, namely x2', but I don't know where to go from here, any hints/help would be appreciated. I am just confused on what to do when one of the variables does not satisfy the positivity constraint, here x2<=10.



      Edit: I think I can rewrite x2<= 10 as x2-10 <= 0. Then introduce x2' = x2-10. Then, can I replace x2 with x2'+10?







      linear-algebra optimization linear-programming






      share|cite|improve this question















      share|cite|improve this question













      share|cite|improve this question




      share|cite|improve this question








      edited Feb 1 at 20:14







      Taffies1

















      asked Feb 1 at 19:58









      Taffies1Taffies1

      52




      52






















          1 Answer
          1






          active

          oldest

          votes


















          0












          $begingroup$

          Replace $x_2$ with $x_2^+ - x_2^-$, with $x_2^+ geq 0$ and $x_2^- geq 0$.






          share|cite|improve this answer









          $endgroup$














            Your Answer





            StackExchange.ifUsing("editor", function () {
            return StackExchange.using("mathjaxEditing", function () {
            StackExchange.MarkdownEditor.creationCallbacks.add(function (editor, postfix) {
            StackExchange.mathjaxEditing.prepareWmdForMathJax(editor, postfix, [["$", "$"], ["\\(","\\)"]]);
            });
            });
            }, "mathjax-editing");

            StackExchange.ready(function() {
            var channelOptions = {
            tags: "".split(" "),
            id: "69"
            };
            initTagRenderer("".split(" "), "".split(" "), channelOptions);

            StackExchange.using("externalEditor", function() {
            // Have to fire editor after snippets, if snippets enabled
            if (StackExchange.settings.snippets.snippetsEnabled) {
            StackExchange.using("snippets", function() {
            createEditor();
            });
            }
            else {
            createEditor();
            }
            });

            function createEditor() {
            StackExchange.prepareEditor({
            heartbeatType: 'answer',
            autoActivateHeartbeat: false,
            convertImagesToLinks: true,
            noModals: true,
            showLowRepImageUploadWarning: true,
            reputationToPostImages: 10,
            bindNavPrevention: true,
            postfix: "",
            imageUploader: {
            brandingHtml: "Powered by u003ca class="icon-imgur-white" href="https://imgur.com/"u003eu003c/au003e",
            contentPolicyHtml: "User contributions licensed under u003ca href="https://creativecommons.org/licenses/by-sa/3.0/"u003ecc by-sa 3.0 with attribution requiredu003c/au003e u003ca href="https://stackoverflow.com/legal/content-policy"u003e(content policy)u003c/au003e",
            allowUrls: true
            },
            noCode: true, onDemand: true,
            discardSelector: ".discard-answer"
            ,immediatelyShowMarkdownHelp:true
            });


            }
            });














            draft saved

            draft discarded


















            StackExchange.ready(
            function () {
            StackExchange.openid.initPostLogin('.new-post-login', 'https%3a%2f%2fmath.stackexchange.com%2fquestions%2f3096662%2flinear-programming-negativity-constraints%23new-answer', 'question_page');
            }
            );

            Post as a guest















            Required, but never shown

























            1 Answer
            1






            active

            oldest

            votes








            1 Answer
            1






            active

            oldest

            votes









            active

            oldest

            votes






            active

            oldest

            votes









            0












            $begingroup$

            Replace $x_2$ with $x_2^+ - x_2^-$, with $x_2^+ geq 0$ and $x_2^- geq 0$.






            share|cite|improve this answer









            $endgroup$


















              0












              $begingroup$

              Replace $x_2$ with $x_2^+ - x_2^-$, with $x_2^+ geq 0$ and $x_2^- geq 0$.






              share|cite|improve this answer









              $endgroup$
















                0












                0








                0





                $begingroup$

                Replace $x_2$ with $x_2^+ - x_2^-$, with $x_2^+ geq 0$ and $x_2^- geq 0$.






                share|cite|improve this answer









                $endgroup$



                Replace $x_2$ with $x_2^+ - x_2^-$, with $x_2^+ geq 0$ and $x_2^- geq 0$.







                share|cite|improve this answer












                share|cite|improve this answer



                share|cite|improve this answer










                answered Feb 1 at 22:34









                LinAlgLinAlg

                10.1k1521




                10.1k1521






























                    draft saved

                    draft discarded




















































                    Thanks for contributing an answer to Mathematics Stack Exchange!


                    • Please be sure to answer the question. Provide details and share your research!

                    But avoid



                    • Asking for help, clarification, or responding to other answers.

                    • Making statements based on opinion; back them up with references or personal experience.


                    Use MathJax to format equations. MathJax reference.


                    To learn more, see our tips on writing great answers.




                    draft saved


                    draft discarded














                    StackExchange.ready(
                    function () {
                    StackExchange.openid.initPostLogin('.new-post-login', 'https%3a%2f%2fmath.stackexchange.com%2fquestions%2f3096662%2flinear-programming-negativity-constraints%23new-answer', 'question_page');
                    }
                    );

                    Post as a guest















                    Required, but never shown





















































                    Required, but never shown














                    Required, but never shown












                    Required, but never shown







                    Required, but never shown

































                    Required, but never shown














                    Required, but never shown












                    Required, but never shown







                    Required, but never shown







                    Popular posts from this blog

                    Can a sorcerer learn a 5th-level spell early by creating spell slots using the Font of Magic feature?

                    ts Property 'filter' does not exist on type '{}'

                    mat-slide-toggle shouldn't change it's state when I click cancel in confirmation window